What Are the Limit Points of the Set {sin(n): n a Positive Integer}?

Click For Summary
The set {sin(n): n a positive integer} has limit points due to the dense nature of the sine function over the interval [-1, 1]. It is suggested that 0, 1, and -1 are potential limit points, as any point within the interval can be approximated closely by elements from the set. The discussion indicates that while sin(n) does not converge, it can still yield an infinite number of limit points. The definition of a limit point is referenced, emphasizing that every neighborhood of a limit point must contain other points from the set. Ultimately, it is concluded that all points in the interval [-1, 1] should be considered limit points of the set.
cragar
Messages
2,546
Reaction score
3

Homework Statement


a)Determine at least three limit points for the set {sin(n): n a positive integer}
b)How many limit points does the set {sin(n): n a positive integer} have?

The Attempt at a Solution


For a it seems that it wouldn't have a limit point because sin(n) would not converge to anything. On the other hand maybe its obvious and maybe 3 limit points would be
sin(1), sin(2), sin(3) , now that I think about it, it seems like a tricky question.
for part b, if sin(n) has 3 limit points then it seems like the set would have an infinite amount of points.
 
Physics news on Phys.org
From http://en.wikipedia.org/wiki/Limit_point we have that x is a limit point of A={sin(n): n a positive integer} if every neighborhood of x has another element of A different from x. So if you can show that you can approximate x as closely as you wish with elements taken from A, then you have a limit point there. I think three natural points to try to show are limit points are 0,1,-1.
 
so really and point in [-1,1] should be a limit point because i can get as close as I want to any point in there with the sin(n), can points in my set {sin(n)} where n is a natural number.
be a limit point?
 
Last edited:
Question: A clock's minute hand has length 4 and its hour hand has length 3. What is the distance between the tips at the moment when it is increasing most rapidly?(Putnam Exam Question) Answer: Making assumption that both the hands moves at constant angular velocities, the answer is ## \sqrt{7} .## But don't you think this assumption is somewhat doubtful and wrong?

Similar threads

  • · Replies 5 ·
Replies
5
Views
2K
Replies
8
Views
2K
Replies
29
Views
5K
  • · Replies 12 ·
Replies
12
Views
2K
  • · Replies 1 ·
Replies
1
Views
2K
Replies
4
Views
2K
  • · Replies 23 ·
Replies
23
Views
3K
  • · Replies 4 ·
Replies
4
Views
20K
  • · Replies 4 ·
Replies
4
Views
2K
  • · Replies 4 ·
Replies
4
Views
2K